Question and Answers Forum

All Questions      Topic List

Geometry Questions

Previous in All Question      Next in All Question      

Previous in Geometry      Next in Geometry      

Question Number 16072 by Tinkutara last updated on 17/Jun/17

Let ABCD be a convex quadrilateral.  Prove that the orthocenters of the  triangles ABC, BCD, CDA and DAB  are the vertices of a quadrilateral  congruent to ABCD and prove that the  centroids of the same triangles are the  vertices of a cyclic quadrilateral.

$$\mathrm{Let}\:{ABCD}\:\mathrm{be}\:\mathrm{a}\:\mathrm{convex}\:\mathrm{quadrilateral}. \\ $$$$\mathrm{Prove}\:\mathrm{that}\:\mathrm{the}\:\mathrm{orthocenters}\:\mathrm{of}\:\mathrm{the} \\ $$$$\mathrm{triangles}\:{ABC},\:{BCD},\:{CDA}\:\mathrm{and}\:{DAB} \\ $$$$\mathrm{are}\:\mathrm{the}\:\mathrm{vertices}\:\mathrm{of}\:\mathrm{a}\:\mathrm{quadrilateral} \\ $$$$\mathrm{congruent}\:\mathrm{to}\:{ABCD}\:\mathrm{and}\:\mathrm{prove}\:\mathrm{that}\:\mathrm{the} \\ $$$$\mathrm{centroids}\:\mathrm{of}\:\mathrm{the}\:\mathrm{same}\:\mathrm{triangles}\:\mathrm{are}\:\mathrm{the} \\ $$$$\mathrm{vertices}\:\mathrm{of}\:\mathrm{a}\:\mathrm{cyclic}\:\mathrm{quadrilateral}. \\ $$

Commented by mrW1 last updated on 06/Jul/17

the question seems to be wrong. I think  the centroids are the vertices of a  quadrilateral congruent to ABCD.    For part 1 please post a figure to  show what is meant.

$$\mathrm{the}\:\mathrm{question}\:\mathrm{seems}\:\mathrm{to}\:\mathrm{be}\:\mathrm{wrong}.\:\mathrm{I}\:\mathrm{think} \\ $$$$\mathrm{the}\:\mathrm{centroids}\:\mathrm{are}\:\mathrm{the}\:\mathrm{vertices}\:\mathrm{of}\:\mathrm{a} \\ $$$$\mathrm{quadrilateral}\:\mathrm{congruent}\:\mathrm{to}\:\mathrm{ABCD}. \\ $$$$ \\ $$$$\mathrm{For}\:\mathrm{part}\:\mathrm{1}\:\mathrm{please}\:\mathrm{post}\:\mathrm{a}\:\mathrm{figure}\:\mathrm{to} \\ $$$$\mathrm{show}\:\mathrm{what}\:\mathrm{is}\:\mathrm{meant}. \\ $$

Commented by mrW1 last updated on 06/Jul/17

yes please.

$$\mathrm{yes}\:\mathrm{please}. \\ $$

Commented by mrW1 last updated on 06/Jul/17

I can prove that the centroids are the vertices of a  quadrilateral congruent to ABCD. Its  area is (1/9) of the area of ABCD.

$$\mathrm{I}\:\mathrm{can}\:\mathrm{prove}\:\mathrm{that}\:\mathrm{the}\:\mathrm{centroids}\:\mathrm{are}\:\mathrm{the}\:\mathrm{vertices}\:\mathrm{of}\:\mathrm{a} \\ $$$$\mathrm{quadrilateral}\:\mathrm{congruent}\:\mathrm{to}\:\mathrm{ABCD}.\:\mathrm{Its} \\ $$$$\mathrm{area}\:\mathrm{is}\:\frac{\mathrm{1}}{\mathrm{9}}\:\mathrm{of}\:\mathrm{the}\:\mathrm{area}\:\mathrm{of}\:\mathrm{ABCD}. \\ $$

Answered by Tinkutara last updated on 07/Jul/17

Commented by Tinkutara last updated on 07/Jul/17

Lemma used:  OA^(→)  + OB^(→)  + OC^(→)  = OH^(→)  and  OH^(→)  = 3OG^(→) .

$$\boldsymbol{\mathrm{Lemma}}\:\boldsymbol{\mathrm{used}}: \\ $$$$\overset{\rightarrow} {{OA}}\:+\:\overset{\rightarrow} {{OB}}\:+\:\overset{\rightarrow} {{OC}}\:=\:\overset{\rightarrow} {{OH}}\:\mathrm{and} \\ $$$$\overset{\rightarrow} {{OH}}\:=\:\mathrm{3}\overset{\rightarrow} {{OG}}. \\ $$

Commented by Tinkutara last updated on 07/Jul/17

Proof:  Let O be circumcenter of the  quadrilateral ABCD and let H_A , H_B ,  H_C , H_D  be the orthocenters of the  triangles BCD, CDA, DAB and  ABC, respectively. Using the lemma,  we have  H_A H_B ^(→)  = OH_B ^(→)  − OH_A ^(→)   = (OC^(→)  + OB^(→)  + OA^(→) ) − (OB^(→)  + OC^(→)  + OD^(→) )  = OA^(→)  − OB^(→)  = BA^(→) ,  and consequently the segments H_A H_B   and AB are parallel and equal in  length. We conclude that the  quadrilaterals ABCD and H_A H_B H_C H_D   are congruent.  For the second claim, let G_A , G_B , G_C   and G_D  denote the centroids. It follows  from the observation above that  G_A G_B G_C G_D  is obtained from  H_A H_B H_C H_D  by a homothetic of center  O and ratio (1/3). Because H_A H_B H_C H_D   is cyclic, the same is true of G_A G_B G_C G_D .  (See Figure 5.27).

$$\boldsymbol{\mathrm{Proof}}: \\ $$$$\mathrm{Let}\:{O}\:\mathrm{be}\:\mathrm{circumcenter}\:\mathrm{of}\:\mathrm{the} \\ $$$$\mathrm{quadrilateral}\:{ABCD}\:\mathrm{and}\:\mathrm{let}\:{H}_{{A}} ,\:{H}_{{B}} , \\ $$$${H}_{{C}} ,\:{H}_{{D}} \:\mathrm{be}\:\mathrm{the}\:\mathrm{orthocenters}\:\mathrm{of}\:\mathrm{the} \\ $$$$\mathrm{triangles}\:{BCD},\:{CDA},\:{DAB}\:\mathrm{and} \\ $$$${ABC},\:\mathrm{respectively}.\:\mathrm{Using}\:\mathrm{the}\:\mathrm{lemma}, \\ $$$$\mathrm{we}\:\mathrm{have} \\ $$$$\overset{\rightarrow} {{H}_{{A}} {H}_{{B}} }\:=\:\overset{\rightarrow} {{OH}_{{B}} }\:−\:\overset{\rightarrow} {{OH}_{{A}} } \\ $$$$=\:\left(\overset{\rightarrow} {{OC}}\:+\:\overset{\rightarrow} {{OB}}\:+\:\overset{\rightarrow} {{OA}}\right)\:−\:\left(\overset{\rightarrow} {{OB}}\:+\:\overset{\rightarrow} {{OC}}\:+\:\overset{\rightarrow} {{OD}}\right) \\ $$$$=\:\overset{\rightarrow} {{OA}}\:−\:\overset{\rightarrow} {{OB}}\:=\:\overset{\rightarrow} {{BA}}, \\ $$$$\mathrm{and}\:\mathrm{consequently}\:\mathrm{the}\:\mathrm{segments}\:{H}_{{A}} {H}_{{B}} \\ $$$$\mathrm{and}\:{AB}\:\mathrm{are}\:\mathrm{parallel}\:\mathrm{and}\:\mathrm{equal}\:\mathrm{in} \\ $$$$\mathrm{length}.\:\mathrm{We}\:\mathrm{conclude}\:\mathrm{that}\:\mathrm{the} \\ $$$$\mathrm{quadrilaterals}\:{ABCD}\:\mathrm{and}\:{H}_{{A}} {H}_{{B}} {H}_{{C}} {H}_{{D}} \\ $$$$\mathrm{are}\:\mathrm{congruent}. \\ $$$$\mathrm{For}\:\mathrm{the}\:\mathrm{second}\:\mathrm{claim},\:\mathrm{let}\:{G}_{{A}} ,\:{G}_{{B}} ,\:{G}_{{C}} \\ $$$$\mathrm{and}\:{G}_{{D}} \:\mathrm{denote}\:\mathrm{the}\:\mathrm{centroids}.\:\mathrm{It}\:\mathrm{follows} \\ $$$$\mathrm{from}\:\mathrm{the}\:\mathrm{observation}\:\mathrm{above}\:\mathrm{that} \\ $$$${G}_{{A}} {G}_{{B}} {G}_{{C}} {G}_{{D}} \:\mathrm{is}\:\mathrm{obtained}\:\mathrm{from} \\ $$$${H}_{{A}} {H}_{{B}} {H}_{{C}} {H}_{{D}} \:\mathrm{by}\:\mathrm{a}\:\mathrm{homothetic}\:\mathrm{of}\:\mathrm{center} \\ $$$${O}\:\mathrm{and}\:\mathrm{ratio}\:\frac{\mathrm{1}}{\mathrm{3}}.\:\mathrm{Because}\:{H}_{{A}} {H}_{{B}} {H}_{{C}} {H}_{{D}} \\ $$$$\mathrm{is}\:\mathrm{cyclic},\:\mathrm{the}\:\mathrm{same}\:\mathrm{is}\:\mathrm{true}\:\mathrm{of}\:{G}_{{A}} {G}_{{B}} {G}_{{C}} {G}_{{D}} . \\ $$$$\left(\mathrm{See}\:\mathrm{Figure}\:\mathrm{5}.\mathrm{27}\right). \\ $$

Commented by Tinkutara last updated on 07/Jul/17

Commented by mrW1 last updated on 07/Jul/17

the question didn′t say that ABCD is  cyclic.

$$\mathrm{the}\:\mathrm{question}\:\mathrm{didn}'\mathrm{t}\:\mathrm{say}\:\mathrm{that}\:\mathrm{ABCD}\:\mathrm{is} \\ $$$$\mathrm{cyclic}. \\ $$

Commented by mrW1 last updated on 07/Jul/17

the answer given doesn′t match the  question. i can not understand.

$$\mathrm{the}\:\mathrm{answer}\:\mathrm{given}\:\mathrm{doesn}'\mathrm{t}\:\mathrm{match}\:\mathrm{the} \\ $$$$\mathrm{question}.\:\mathrm{i}\:\mathrm{can}\:\mathrm{not}\:\mathrm{understand}. \\ $$

Commented by mrW1 last updated on 07/Jul/17

i have said the question is not correct.  so i can not give a solution to it.   the proof i meant is following. but it  is not the proof to your question exactly.

$$\mathrm{i}\:\mathrm{have}\:\mathrm{said}\:\mathrm{the}\:\mathrm{question}\:\mathrm{is}\:\mathrm{not}\:\mathrm{correct}. \\ $$$$\mathrm{so}\:\mathrm{i}\:\mathrm{can}\:\mathrm{not}\:\mathrm{give}\:\mathrm{a}\:\mathrm{solution}\:\mathrm{to}\:\mathrm{it}.\: \\ $$$$\mathrm{the}\:\mathrm{proof}\:\mathrm{i}\:\mathrm{meant}\:\mathrm{is}\:\mathrm{following}.\:\mathrm{but}\:\mathrm{it} \\ $$$$\mathrm{is}\:\mathrm{not}\:\mathrm{the}\:\mathrm{proof}\:\mathrm{to}\:\mathrm{your}\:\mathrm{question}\:\mathrm{exactly}. \\ $$

Commented by mrW1 last updated on 07/Jul/17

Commented by mrW1 last updated on 07/Jul/17

ABCD is an any convex quadrilateral.  K,L,M,N are the centroids described.  ((GM)/(GB))=(1/3)  ((GN)/(GC))=(1/3)  ⇒MN//BC and MN=((BC)/3)  similarly  NL//AB and NL=((AB)/3)  LK//AD and LK=((AD)/3)  KM//DC and KM=((DC)/3)  ⇒LKNM is congruent to ABCD  ⇒Area [KLNM]=(1/9) Area [ABCD]

$$\mathrm{ABCD}\:\mathrm{is}\:\mathrm{an}\:\mathrm{any}\:\mathrm{convex}\:\mathrm{quadrilateral}. \\ $$$$\mathrm{K},\mathrm{L},\mathrm{M},\mathrm{N}\:\mathrm{are}\:\mathrm{the}\:\mathrm{centroids}\:\mathrm{described}. \\ $$$$\frac{\mathrm{GM}}{\mathrm{GB}}=\frac{\mathrm{1}}{\mathrm{3}} \\ $$$$\frac{\mathrm{GN}}{\mathrm{GC}}=\frac{\mathrm{1}}{\mathrm{3}} \\ $$$$\Rightarrow\mathrm{MN}//\mathrm{BC}\:\mathrm{and}\:\mathrm{MN}=\frac{\mathrm{BC}}{\mathrm{3}} \\ $$$$\mathrm{similarly} \\ $$$$\mathrm{NL}//\mathrm{AB}\:\mathrm{and}\:\mathrm{NL}=\frac{\mathrm{AB}}{\mathrm{3}} \\ $$$$\mathrm{LK}//\mathrm{AD}\:\mathrm{and}\:\mathrm{LK}=\frac{\mathrm{AD}}{\mathrm{3}} \\ $$$$\mathrm{KM}//\mathrm{DC}\:\mathrm{and}\:\mathrm{KM}=\frac{\mathrm{DC}}{\mathrm{3}} \\ $$$$\Rightarrow\mathrm{LKNM}\:\mathrm{is}\:\mathrm{congruent}\:\mathrm{to}\:\mathrm{ABCD} \\ $$$$\Rightarrow\mathrm{Area}\:\left[\mathrm{KLNM}\right]=\frac{\mathrm{1}}{\mathrm{9}}\:\mathrm{Area}\:\left[\mathrm{ABCD}\right] \\ $$

Commented by mrW1 last updated on 07/Jul/17

this proof is better than that given in  your book, i think.  ABCD must not be cyclic!

$$\mathrm{this}\:\mathrm{proof}\:\mathrm{is}\:\mathrm{better}\:\mathrm{than}\:\mathrm{that}\:\mathrm{given}\:\mathrm{in} \\ $$$$\mathrm{your}\:\mathrm{book},\:\mathrm{i}\:\mathrm{think}. \\ $$$$\mathrm{ABCD}\:\mathrm{must}\:\mathrm{not}\:\mathrm{be}\:\mathrm{cyclic}! \\ $$

Commented by Tinkutara last updated on 08/Jul/17

Thanks Sir!

$$\mathrm{Thanks}\:\mathrm{Sir}! \\ $$

Terms of Service

Privacy Policy

Contact: info@tinkutara.com